(37 intermediate revisions by 2 users not shown)
Line 1: Line 1:
= [[ECE PhD Qualifying Exams|ECE Ph.D. Qualifying Exam]] in "Communication, Networks, Signal, and Image Processing" (CS)  =
+
[[Category:ECE]]
 +
[[Category:QE]]
 +
[[Category:CNSIP]]
 +
[[Category:problem solving]]
 +
[[Category:image processing]]
  
= [[ECE-QE_CS5-2011|Question 1, August 2011]], Part 1 =
+
<center>
 +
<font size= 4>
 +
[[ECE_PhD_Qualifying_Exams|ECE Ph.D. Qualifying Exam]]
 +
</font size>
  
:[[ECE-QE_CS5-2011_solusion-1|Part 1]],[[ECE-QE CS1-2011 solusion-2|2]]]
+
<font size= 4>
 +
Communication, Networking, Signal and Image Processing (CS)
 +
 
 +
Question 5: Image Processing
 +
</font size>
 +
 
 +
August 2011
 +
</center>
 +
----
 +
----
 +
=Part 2=
 +
Jump to [[ECE-QE_CS5-2011_solusion-1|Part 1]],[[ECE-QE CS1-2011 solusion-2|2]]
  
 
----
 
----
Line 39: Line 57:
  
 
<math>
 
<math>
P_{\theta}(\rho)  = \int_{-\infty}^{+\infty}{\delta(r cos\theta - z sin\theta, r sin\theta + z cos \theta) dz}
+
p_{\theta}(r)  = \int_{-\infty}^{+\infty}{\delta(r cos\theta - z sin\theta, r sin\theta + z cos \theta) dz}
 
</math>
 
</math>
  
Line 45: Line 63:
 
<math>
 
<math>
 
= z \text{ when } \left\{\begin{matrix}
 
= z \text{ when } \left\{\begin{matrix}
r cos\theta - z sin\theta = 0  
+
r cos \theta - z sin \theta = 0
\\
+
\\  
 
r sin \theta + z cos \theta = 0
 
r sin \theta + z cos \theta = 0
 
\end{matrix}\right.
 
\end{matrix}\right.
 
</math>
 
</math>
 +
  
 
<math>
 
<math>
 
= \frac{r cos\theta}{sin \theta}, \theta > 0
 
= \frac{r cos\theta}{sin \theta}, \theta > 0
 +
</math>
 +
 +
<math>{\color{red}
 +
\text{This answer is incorrect. The correct answer is as following:}}
 +
</math>
 +
 +
<math>{\color{green}
 +
\text{Recall:}}
 +
</math>
 +
 +
 +
<math>{\color{green}
 +
\text{i) } \int_{-\infty}^{+\infty}{f(g(t)) \delta (t) dt} = f(g(t=0)) \int_{-\infty}^{+\infty}{\delta (t) dt}
 +
}</math>
 +
 +
 +
<math>{\color{green}
 +
\text{ii) } \int_{-\infty}^{+\infty}{\delta (\alpha t) dt} = \int_{-\infty}^{+\infty}{\delta (u) \frac{du}{|\alpha|}} = \frac{1}{|\alpha|}
 +
}</math>
 +
 +
 +
<math>{\color{green}
 +
\text{iii) } \delta() \text{ function is separable: } \delta(x,y) = \delta(x) \cdot \delta(y)
 +
}</math>
 +
 +
 +
<font face="serif"><span style="font-size: 19px;"><math>\color{green}
 +
\text{ Define } u = r cos\theta - z sin\theta
 +
</math></span></font>
 +
 +
 +
<math>\color{green}
 +
\Rightarrow dz = \frac{du}{|sin\theta|}
 +
</math>
 +
 +
 +
<font face="serif"><span style="font-size: 19px;"><math>\color{green}
 +
\text{ Now }
 +
</math></span></font>
 +
 +
 +
<math>\color{green}
 +
p_{\theta}(r)  = \int_{-\infty}^{+\infty}{\delta(r cos\theta - z sin\theta, r sin\theta + z cos \theta) dz}
 +
</math>
 +
 +
 +
<math>\color{green}
 +
p_{\theta}(r)  = \int_{-\infty}^{+\infty}{\delta(g(u)) \delta(u) \frac{du}{|sin\theta|}} = \frac{\delta(u=0)}{|sin\theta|}
 +
</math>
 +
 +
 +
<math>\color{green}
 +
= \frac{\delta(\frac{r}{sin\theta})}{|sin\theta|} = \frac{|sin\theta|}{|sin\theta|} \delta(r) = \delta(r)
 
</math>
 
</math>
  
Line 59: Line 131:
 
<math>\color{blue}\text{Solution 2:}</math>  
 
<math>\color{blue}\text{Solution 2:}</math>  
  
[[Image:.jpg]]
+
[[Image:.QE_11_CS5_2_a_sol2.PNG]]
  
 
<math>
 
<math>
P_{\theta}(\rho)  = \int_{-\infty}^{+\infty}{\delta(r cos\theta - z sin\theta, r sin\theta + z cos \theta) dz}
+
p_{\theta}(r)  = \int_{-\infty}^{+\infty}{\delta(r cos\theta - z sin\theta, r sin\theta + z cos \theta) dz}
 
</math>
 
</math>
  
<math>
+
 
 +
<font face="serif"><span style="font-size: 19px;"><math>
 
= \delta(r)
 
= \delta(r)
</math>
+
</math></span></font>
 +
 
 +
 
 +
<font face="serif"><span style="font-size: 19px;"><math>{\color{green}
 +
\text{Here, the student uses the intuitive solution: in this case the answer does not depend on } \theta \text{, since the image just contains a peak at origin. }
 +
}</math></span></font>
  
 
----
 
----
Line 77: Line 155:
  
 
<math>
 
<math>
P_{\theta}(\rho)  = \int_{-\infty}^{+\infty}{\delta(r cos\theta - z sin\theta - 1, r sin\theta + z cos \theta - 1) dz}
+
p_{\theta}(r)  = \int_{-\infty}^{+\infty}{\delta(r cos\theta - z sin\theta - 1, r sin\theta + z cos \theta - 1) dz}
 
</math>
 
</math>
 
  
 
<math>
 
<math>
 
= z \text{ when } \left\{\begin{matrix}
 
= z \text{ when } \left\{\begin{matrix}
r cos\theta - z sin\theta = 1  
+
r cos \theta - z sin \theta = 1
\\
+
\\  
 
r sin \theta + z cos \theta = 1
 
r sin \theta + z cos \theta = 1
 
\end{matrix}\right.
 
\end{matrix}\right.
 
</math>
 
</math>
 +
  
 
<math>
 
<math>
Line 93: Line 171:
 
</math>
 
</math>
  
 +
<math>{\color{red}
 +
\text{This answer is incorrect. The correct answer is as following:}}
 +
</math>
 +
 +
<font face="serif"><span style="font-size: 19px;"><math>\color{green}
 +
\text{ Similar to the solution 1 to part a) we define u: } u = r cos\theta - z sin\theta - 1
 +
</math></span></font>
 +
 +
 +
<font face="serif"><span style="font-size: 19px;"><math>\color{green}
 +
\text{ Following the same logic as in part a) we obtain the final answer:}
 +
</math></span></font>
 +
 +
 +
<math>\color{green}
 +
p_{\theta}(r)  = \delta(r - (cos\theta + sin \theta)) = \delta(r - \sqrt{2} cos (\theta - \frac{\pi}{4}))
 +
</math>
  
 
----
 
----
Line 98: Line 193:
 
<math>\color{blue}\text{Solution 2:}</math><br>  
 
<math>\color{blue}\text{Solution 2:}</math><br>  
  
[[Image:.jpg]]
+
[[Image:QE_11_CS5_2_b_sol2.PNG]]
  
 
<math>
 
<math>
\tilde{p}_\theta(r) = p_{\theta}(r - \sqrt{1+1} cos(\theta - tan^{-1}\frac{1}{1}))
+
\tilde{p}_\theta(r) = p_{\theta}(r - \sqrt{1+1} cos(\theta - tan^{-1}(\frac{1}{1})))
 
</math>
 
</math>
 +
  
 
<math>
 
<math>
= p_\theta(r - \sqrt{2} cos(\theta - \frac{\pi}{4})
+
= p_\theta(r - \sqrt{2} cos(\theta - \frac{\pi}{4}))
 
</math>
 
</math>
 +
  
 
<math>
 
<math>
= \delta(r - \sqrt{2} cos(\theta - \frac{\pi}{4})
+
= \delta(r - \sqrt{2} cos(\theta - \frac{\pi}{4}))
 
</math>
 
</math>
 +
 +
 +
<font face="serif"><span style="font-size: 19px;"><math>{\color{green}
 +
\text{Again, the student uses the intuitive solution: in this case the answer does depend on } \theta \text{, since the peak is shifted from the origin to the point } (1,1).
 +
}</math></span></font>
  
 
----
 
----
Line 123: Line 225:
  
 
<math>
 
<math>
P_{\theta}(\rho)  = \int_{-\infty}^{+\infty}{rect(\sqrt{(r cos\theta - z sin\theta)^2, (r sin\theta + z cos \theta)^2) dz}
+
p_{\theta}(r)  = \int_{-\infty}^{+\infty}{rect(\sqrt{(r cos\theta - z sin\theta)^2 + (r sin\theta + z cos \theta)^2)} dz}
 
</math>
 
</math>
 +
 +
 +
<math>\color{green}
 +
\text{Recall should be added:}
 +
</math>
 +
 +
<math>\color{green}
 +
rect(t) = \left\{\begin{matrix}
 +
1, for |t|\leq \frac{1}{2}
 +
\\
 +
0, otherwise
 +
\end{matrix}\right.
 +
</math>
 +
 +
 +
<font face="serif"><span style="font-size: 19px;"><math>\color{green}
 +
\text{therefore: }
 +
</math></span></font>
  
  
 
<math>
 
<math>
= \int_{-\sqrt{\frac{1}{4} - r^2}}^{\sqrt{\frac{1}{4} - r^2}}{1 dz}
+
p_{\theta}(r)  = \int_{-\sqrt{\frac{1}{4} - r^2}}^{\sqrt{\frac{1}{4} - r^2}}{1 dz}
 
</math>
 
</math>
 +
  
 
<math>
 
<math>
 
= \left\{\begin{matrix}
 
= \left\{\begin{matrix}
&\sqrt{1 - 4r^2}, &\text{ if }|r| \leq \frac{1}{2}
+
\sqrt{1 - 4r^2}, &\text{ if }|r| \leq \frac{1}{2}
 
\\
 
\\
&0, &\text{ otherwise}
+
0, &\text{ otherwise}
 
\end{matrix}\right.
 
\end{matrix}\right.
 
</math>
 
</math>
 
  
 
----
 
----
Line 144: Line 264:
 
<math>\color{blue}\text{Solution 2:}</math><br>  
 
<math>\color{blue}\text{Solution 2:}</math><br>  
  
sol2 here
+
[[Image:QE_11_CS5_2_c_sol2.PNG]]
 +
 
 +
<math>
 +
p_{\theta}(r)  = \int_{-\infty}^{+\infty}{f(r cos\theta - z sin\theta, r sin\theta + z cos \theta) dz}
 +
</math>
 +
 
 +
 
 +
<math>
 +
= \int_{-\sqrt{\frac{1}{4} - r^2}}^{\sqrt{\frac{1}{4} - r^2}}{1 dz} = \sqrt{1 - 4r^2}, \text{ if }|r| \leq \frac{1}{2}
 +
</math>
 +
 
 +
 
 +
<font face="serif"><span style="font-size: 19px;"><math>
 +
\text{ else } p_{\theta}(r) = 0
 +
</math></span></font>
 +
 
 
----
 
----
  
Line 154: Line 289:
  
 
<math>
 
<math>
P_{\theta}(\rho)  = \int_{-\infty}^{+\infty}{rect(\sqrt{(r cos\theta - z sin\theta - 1)^2, (r sin\theta + z cos \theta - 1)^2) dz}
+
p_{\theta}(r)  = \int_{-\infty}^{+\infty}{rect \left( \sqrt{(r cos\theta - z sin\theta - 1)^2 + (r sin\theta + z cos \theta - 1)^2} \right) dz}
 
</math>
 
</math>
  
Line 161: Line 296:
 
= \int_{-\sqrt{\frac{1}{4} - (r - (cos\theta + sin\theta))^2}}^{\sqrt{\frac{1}{4} - (r - (cos\theta + sin\theta))^2}}{1 dz}
 
= \int_{-\sqrt{\frac{1}{4} - (r - (cos\theta + sin\theta))^2}}^{\sqrt{\frac{1}{4} - (r - (cos\theta + sin\theta))^2}}{1 dz}
 
</math>
 
</math>
 +
  
 
<math>
 
<math>
 
= \left\{\begin{matrix}
 
= \left\{\begin{matrix}
&\sqrt{1 - 4(r - (cos\theta + sin\theta))^2}, &\text{ if }|r| \leq \frac{1}{2}
+
\sqrt{1 - 4(r - (cos\theta + sin\theta))^2}, &\text{ if }|r| \leq \frac{1}{2}
 
\\
 
\\
&0, &\text{ otherwise}
+
0, &\text{ otherwise}
 +
\end{matrix}\right.
 +
</math>
 +
 
 +
 
 +
<math>\color{green}
 +
\text{ To make it more clear, the following form could be obtained:}
 +
</math>
 +
 
 +
<math>\color{green}
 +
= \left\{\begin{matrix}
 +
\sqrt{1 - 4(r - \sqrt{2} cos (\theta - \frac{\pi}{4}))^2}, &\text{ if }|r| \leq \frac{1}{2}
 +
\\
 +
0, &\text{ otherwise}
 
\end{matrix}\right.
 
\end{matrix}\right.
 
</math>
 
</math>
Line 174: Line 323:
 
<math>\color{blue}\text{Solution 2:}</math><br>  
 
<math>\color{blue}\text{Solution 2:}</math><br>  
  
sol2 here
+
<math>
 +
\tilde{p}_\theta(r) = p_{\theta}(r - \sqrt{1+1} cos(\theta - tan^{-1}(\frac{1}{1})))
 +
</math>
 +
 
 +
 
 +
<math>
 +
= p_\theta(r - \sqrt{2} cos(\theta - \frac{\pi}{4}))
 +
</math>
 +
 
 +
 
 +
<math>
 +
\text{ where } p_\theta(r) = \left\{\begin{matrix}
 +
\sqrt{1 - 4r^2}, &\text{ if }|r| \leq \frac{1}{2}
 +
\\
 +
0, &\text{ else}
 +
\end{matrix}\right.
 +
</math>
 +
 
 +
 
 +
<font face="serif"><span style="font-size: 19px;"><math>{\color{green}
 +
\text{Here, the student uses the results from solutions to part b and c.}
 +
}</math></span></font>
 +
 
 
----
 
----
 
<math>\color{blue}\text{e) Describe in precise detail, the steps required to perform filtered back projection (FBP) reconstruction of } f(x,y).
 
<math>\color{blue}\text{e) Describe in precise detail, the steps required to perform filtered back projection (FBP) reconstruction of } f(x,y).
Line 182: Line 353:
 
<math>\color{blue}\text{Solution 1:}</math>  
 
<math>\color{blue}\text{Solution 1:}</math>  
  
<math>
+
<font face="serif"><span style="font-size: 19px;"><math>
1. \text{ Compute } \rho_{\theta}(r).
+
1. \text{ Compute } \rho_{\theta}(r)
</math>  
+
</math></span></font>  
  
<math>
+
 
 +
<font face="serif"><span style="font-size: 19px;"><math>
 
2. \text{ Compute FT of step 1.}
 
2. \text{ Compute FT of step 1.}
</math>  
+
</math></span></font>  
  
<math>
+
<font face="serif"><span style="font-size: 19px;"><math>
3. \text{ Multiply step 2 by the filter } H(\rho) = f_c[rect(\frac{f}{2f_c}) - \Lambda(\frac{f}{f_c})].
+
3. \text{ Multiply step 2 by the filter } H(\rho) = |\rho| = f_c \left [ rect(\frac{f}{2f_c}) - \Lambda(\frac{f}{f_c}) \right ]
 +
</math></span></font>
 +
 
 +
<font face="serif"><span style="font-size: 19px;"><math>
 +
4. \text{ Compute inverseFT of step 3.}
 +
</math></span></font> 
 +
 
 +
<math> \color{red}
 +
\text{ This answer is not complete. We need the final step:}
 
</math>
 
</math>
 +
 +
 +
<font face="serif"><span style="font-size: 19px;"><math>\color{red}
 +
5. \text{ Back project } g_{\theta}(r) \text{ (obtained from step 4) and get: }
 +
</math></span></font>
 
   
 
   
 +
<font face="serif"><span style="font-size: 19px;"><math>\color{red}
 +
f(x,y) = \int_{0}^{\pi}{g_\theta(xcos\theta + ysin\theta)d\theta}
 +
</math></span></font>
 +
 +
----
 +
 +
<math>\color{blue}\text{Solution 2:}</math><br>
 +
 +
<font face="serif"><span style="font-size: 19px;"><math>
 +
1. \text{ Measure the projections } \rho_{\theta}(r) \text{ at various angles}
 +
</math></span></font> 
 +
 +
<font face="serif"><span style="font-size: 19px;"><math>
 +
2. \text{ Filter the projections } \rho_{\theta}(r) \text{ with } h(r) \text{, where } H(\rho) = |\rho| \text{ and get } g_{\theta}(r)
 +
</math></span></font> 
 +
 +
<font face="serif"><span style="font-size: 19px;"><math>
 +
3. \text{ Back project } g_{\theta}(r) \text{ along } r = xcos\theta + ysin\theta \text{ and get }
 +
</math></span></font>
 +
 +
<font face="serif"><span style="font-size: 19px;"><math>
 +
f(x,y) = \int_{0}^{\pi}{g_\theta(xcos\theta + ysin\theta)d\theta}
 +
</math></span></font>
 +
 +
----
 +
=Solution 3:=
 +
 +
a)
 +
 +
<math>p_\theta(r)=\begin{cases}1, & \text{if } r=0\\0, & \text{otherwisr}\end{cases}=\delta(r)</math>.
 +
 +
[[Image:CW_QE_2011_CS5_2_a_sol3.png|200px]]
 +
 +
 +
b)
 +
 +
* If <math>0\leq\theta<\frac{\pi}{4}</math>, <math>d=\sqrt{2}\text{cos}(\frac{\pi}{4}-\theta)=\sqrt{2}[\text{cos}\frac{\pi}{4}\text{cos}\theta+\text{sin}\frac{\pi}{4}\text{sin}\theta]=\text{cos}\theta+\text{sin}\theta</math>.
 +
 +
&nbsp;&nbsp;&nbsp;&nbsp;&nbsp;&nbsp;&nbsp;&nbsp;[[Image:CW_QE_2011_CS5_2_b_1_sol3.png|300px]]
 +
 +
* If <math>\frac{\pi}{4}\leq \theta < \frac{\pi}{2}</math>, <math>d=\sqrt{2}\text{cos}(\theta-\frac{\pi}{4})=\sqrt{2}\text{cos}(\frac{\pi}{4}-\theta)=\text{cos}\theta+\text{sin}\theta</math>.
 +
 +
&nbsp;&nbsp;&nbsp;&nbsp;&nbsp;&nbsp;&nbsp;&nbsp;[[Image:CW_QE_2011_CS5_2_b_2_sol3.png|300px]]
 +
 +
* If <math>\frac{\pi}{2}\leq \theta < \frac{3\pi}{4}</math>, <math>d=\sqrt{2}\text{sin}(\frac{\pi}{4}-(\theta-\frac{\pi}{2}))=\sqrt{2}\text{sin}(\frac{3\pi}{4}-\theta)=\sqrt{2}[\text{sin}\frac{3\pi}{4}\text{cos}\theta-\text{cos}\frac{3\pi}{4}\text{sin}\theta]=\text{cos}\theta+\text{sin}\theta</math>.
 +
 +
&nbsp;&nbsp;&nbsp;&nbsp;&nbsp;&nbsp;&nbsp;&nbsp;[[Image:CW_QE_2011_CS5_2_b_3_sol3.png|300px]]
 +
 +
* If <math>\frac{3\pi}{4}\leq \theta < 2\pi</math>, <math>d=-\sqrt{2}\text{sin}(\theta-\frac{\pi}{2}-\frac{\pi}{4})=-\sqrt{2}\text{sin}(\theta-\frac{3\pi}{4})=-\sqrt{2}[\text{sin}\theta\text{cos}\frac{3\pi}{4}-\text{cos}\theta\text{sin}\frac{3\pi}{4}]=\text{cos}\theta+\text{sin}\theta</math>.
 +
 +
&nbsp;&nbsp;&nbsp;&nbsp;&nbsp;&nbsp;&nbsp;&nbsp;[[Image:CW_QE_2011_CS5_2_b_4_sol3.png|300px]]
 +
 +
<math>\Rightarrow p_\theta(r)=\delta(r-(\text{cos}\theta+\text{sin}\theta))</math> for all <math>\theta</math>.
 +
 +
 +
c)
 +
 +
Since <math>f(x,y)</math> is symmetric to <math>(0,0)</math>
 +
 +
<math>\Rightarrow p_\theta(r)=p_0(r)=\int_{-\sqrt{\frac{1}{4}-r^2}}^{\sqrt{\frac{1}{4}-r^2}}1\cdot dz=2\int_0^{\sqrt{\frac{1}{4}-r^2}}1\cdot dz\text{, for }\vert r\vert<\frac{1}{2}</math>
 +
 +
[[Image:CW_QE_2011_CS5_2_c_sol3.png|200px]]
 +
 +
<math>\Rightarrow p_\theta(r)=2\sqrt{\frac{1}{4}-r^2}\text{rect}(r)</math>
 +
 +
 +
d)
 +
 +
From part (b), we know that, if <math>f(x,y)</math> has <math>(1,1)</math> shift in <math>x-y</math> plane, the <math>p_\theta(r)</math> would have <math>(\text{cos}\theta+\text{sin}\theta)</math> shift in <math>r</math>-direction.
 +
 +
<math>\Rightarrow p_\theta(r)=2\sqrt{\frac{1}{4}-(r-(\text{cos}\theta+\text{sin}\theta))^2}\text{rect}(r-(\text{cos}\theta+\text{sin}\theta))</math>
 +
 +
 +
e)
 +
 +
* First, measure the forward projection <math>p_\theta(r)</math> for sufficient angles <math>\theta s</math>.
 +
* Second, filter the forward projection with <math>h(r): g_\theta(r)=h(r)\ast p_\theta(r)</math>, where <math>h(r)=\text{CTFT}^{-1}\left\{\vert\rho\vert\right\}</math>.
 +
* Finally, back project filtered projection to reconstruct <math>f(x,y)</math>. <math>f(x,y)=\int_0^\pi g_\theta(x\text{cos}\theta+y\text{sin}\theta)d\theta</math>.
 +
 +
 +
 +
=Solution 4:=
 +
 +
a)
 +
 +
<math>P_\theta(\rho)=F(\rho\text{cos}\theta, \rho\text{sin}\theta)</math>
 +
 +
When <math>f(x,y)=\delta(x,y)</math>, <math>F(\mu,\nu)=1</math>.
 +
 +
So <math>p_\theta(r)=\delta_\theta(r)</math>.
 +
 +
<font color="green"><u>'''Comments'''</u>:
 +
 +
* The answer would be easier to understand if the last statement was expressed in this way: <math>p_{\theta}(r)=\mathcal{F}^{-1}\{ P_{\theta}(\rho)\}=\mathcal{F}^{-1}\{1\}=\delta_{\theta}(r)</math>, where <math>\mathcal{F}^{-1}\{\cdot\}</math> is the Fourier inverse transform.
 +
 +
</font>
 +
 +
 +
b)
 +
 +
When <math>f(x,y)=\delta(x-1,y-1)</math>, <math>F(\mu,\nu)=e^{-j\mu}\cdot e^{-j\nu}</math>.
 +
 +
So <math>P_\theta(\rho)=e^{-j\rho\text{cos}\theta}\cdot e^{-j\rho\text{sin}\theta}</math>, <math>p_\theta(r)=\delta(r-\text{cos}\theta-\text{sin}\theta)</math>.
 +
 +
 +
c)
 +
 +
[[Image:CW_QE_2011_CS5_2_c_sol4.png|200px]]
 +
 +
<math>p_\theta(r)=\int_{-\sqrt{\frac{1}{4}-r^2}}^{\sqrt{\frac{1}{4}-r^2}}1dz=\sqrt{1-4r^2}</math>
 +
 +
So <math>p_\theta(r)=\begin{cases} \sqrt{1-4r^2}, & |r|<\frac{1}{2}\\0, & |r|>\frac{1}{2}\end{cases}</math>.
 +
 +
 +
d)
 +
 +
[[Image:CW_QE_2011_CS5_2_d_sol4.png|300px]]
 +
 +
<math>r_0=\sqrt{2}\cdot\left(\frac{\pi}{4}-\theta\right)</math>
 +
 +
<math>p_\theta(r)=\int_{-\infty}^{\infty}f(r\text{cos}\theta-z\text{sin}\theta, r\text{sin}\theta+z\text{cos}\theta)dz=\int_{-\sqrt{\frac{1}{4}-(r-r_0)^2}}^{\sqrt{\frac{1}{4}-(r-r_0)^2}}adz</math>
 +
 +
So <math>p_\theta(r)=\begin{cases}\sqrt{1-4\left(r-\sqrt{2}\left(\frac{\pi}{4}-\theta\right)\right)}, & |r-r_0|\leq\frac{1}{2}\\0, & \text{otherwise}\end{cases}</math>.
 +
 +
<font color="green"><u>'''Comments'''</u>:
 +
 +
* The computation for <math>r_0</math> is wrong. The correct expression of <math>r_0</math> should be: <math>r_0 = \sqrt{2}\sin(\frac{\pi}{4}-\theta)</math>. It can also be expressed as: <math>r_0=\cos\theta+\sin\theta</math>.
 +
* The rest statement is correct, and then the final result becomes: <math>p_{\theta}(r)=\begin{cases}\sqrt{1-4(r-r_0)}, & |r-r_0|\leq\frac{1}{2}\\0, & \text{otherwise} \end{cases}</math>.
 +
 +
</font>
 +
 +
 +
e)
 +
 
<math>
 
<math>
4. \text{ Compute inverseFT of step 3.}
+
\begin{align}
 +
f(x,y) & =\int_{-\infty}^{\infty}\int_{-\infty}^{\infty}F(\mu,\nu)e^{2\pi j(x\mu+y\nu)}d\mu d\nu\\
 +
& = \int_{-\infty}^{\infty}\int_{0}^{\pi}p_\theta(\rho)e^{2\pi j(x\rho\text{cos}\theta+y\rho\text{sin}\theta)}|\rho|d\theta d\rho\\
 +
& = \int_{0}^{\pi}\underbrace{\int_{-\infty}^{\infty}|\rho|p_\theta(\rho)e^{2\pi j(x\rho\text{cos}\theta+y\rho\text{sin}\theta)}d\rho}_{g_\theta(x\text{cos}\theta+y\text{sin}\theta)}d\theta
 +
\end{align}
 
</math>
 
</math>
 +
 +
So <math>g_\theta(t)</math> is given by:
 +
 +
<math>g_\theta(t)=\int_{-\infty}^{\infty}|\rho|p_\theta(\rho)e^{2\pi j\rho t}d\rho=\text{CTFT}^{-1}\{|\rho|p_\theta(\rho)\}=h(t)\ast p_\theta(r)</math>
 +
 +
So what we do is:
 +
 +
# measure projection <math>p_\theta(r)</math>;
 +
# filter the projection <math>g_\theta(r)=h(r)\ast p_\theta(r)</math>;
 +
# back project filtered projection: <math>f(x,y)=\int_0^{\pi}g_\theta(x\text{cos}\theta)+y\text{sin}\theta)d\theta</math>.
  
  
 
----
 
----
 +
=Related Problems=
  
<math>\color{blue}\text{Solution 2:}</math><br>  
+
a) Calculate the forward projection <math>p_\theta(r)</math>, for <math>f(x,y)=\text{rect}(x,y)</math>, as <math>\theta=0</math> and <math>\theta=\pi/4</math>.
 +
 
 +
b) Calculate the forward projection <math>p_\theta(r)</math>, for <math>f(x,y)=\delta(x-a,y-b)</math>.
  
sol2 here
 
 
----
 
----
  
Line 215: Line 550:
 
----
 
----
  
[[ECE PhD Qualifying Exams|Back to ECE Qualifying Exams (QE) page]]
+
[[ECE PhD Qualifying Exams|Back to ECE Qualifying Exams (QE) page]]
 
+
[[Category:ECE]] [[Category:QE]] [[Category:Automatic_Control]] [[Category:Problem_solving]]
+

Latest revision as of 01:51, 31 March 2015


ECE Ph.D. Qualifying Exam

Communication, Networking, Signal and Image Processing (CS)

Question 5: Image Processing

August 2011



Part 2

Jump to Part 1,2


 $ \color{blue}\text{Consider an image } f(x,y) \text{ with a forward projection} $

                $ \color{blue} p_{\theta}(r) = \mathcal{FP}\left \{ f(x,y) \right \} $

                             $ \color{blue} = \int_{-\infty}^{\infty}{f \left ( r cos(\theta) - z sin(\theta),r sin(\theta) + z cos(\theta) \right )dz}. $

$ \color{blue} \text{Let } F(\mu,\nu) \text{ be the continuous-time Fourier transform of } f(x,y) \text{ given by} $
              $ \color{blue} F(u,v) = \int_{-\infty}^{\infty}{\int_{-\infty}^{\infty}{f(x,y)e^{-j2\pi(ux,vy)}dx}dy} $

$ \color{blue} \text{and let } P_{\theta}(\rho) \text{ be the continuous-time Fourier transform of } p_{\theta}(r) \text{ given by} $
              $ \color{blue} P_{\theta}(\rho) = \int_{-\infty}^{\infty}{p_{\theta}(r)e^{-j2\pi(\rho r)}dr}. $


$ \color{blue}\text{a) Calculate the forward projection }p_{\theta}(r) \text{, for } f(x,y) = \delta(x,y). $

$ \color{blue}\text{Solution 1:} $

$ p_{\theta}(r) = \int_{-\infty}^{+\infty}{\delta(r cos\theta - z sin\theta, r sin\theta + z cos \theta) dz} $


$ = z \text{ when } \left\{\begin{matrix} r cos \theta - z sin \theta = 0 \\ r sin \theta + z cos \theta = 0 \end{matrix}\right. $


$ = \frac{r cos\theta}{sin \theta}, \theta > 0 $

$ {\color{red} \text{This answer is incorrect. The correct answer is as following:}} $

$ {\color{green} \text{Recall:}} $


$ {\color{green} \text{i) } \int_{-\infty}^{+\infty}{f(g(t)) \delta (t) dt} = f(g(t=0)) \int_{-\infty}^{+\infty}{\delta (t) dt} } $


$ {\color{green} \text{ii) } \int_{-\infty}^{+\infty}{\delta (\alpha t) dt} = \int_{-\infty}^{+\infty}{\delta (u) \frac{du}{|\alpha|}} = \frac{1}{|\alpha|} } $


$ {\color{green} \text{iii) } \delta() \text{ function is separable: } \delta(x,y) = \delta(x) \cdot \delta(y) } $


$ \color{green} \text{ Define } u = r cos\theta - z sin\theta $


$ \color{green} \Rightarrow dz = \frac{du}{|sin\theta|} $


$ \color{green} \text{ Now } $


$ \color{green} p_{\theta}(r) = \int_{-\infty}^{+\infty}{\delta(r cos\theta - z sin\theta, r sin\theta + z cos \theta) dz} $


$ \color{green} p_{\theta}(r) = \int_{-\infty}^{+\infty}{\delta(g(u)) \delta(u) \frac{du}{|sin\theta|}} = \frac{\delta(u=0)}{|sin\theta|} $


$ \color{green} = \frac{\delta(\frac{r}{sin\theta})}{|sin\theta|} = \frac{|sin\theta|}{|sin\theta|} \delta(r) = \delta(r) $


$ \color{blue}\text{Solution 2:} $

.QE 11 CS5 2 a sol2.PNG

$ p_{\theta}(r) = \int_{-\infty}^{+\infty}{\delta(r cos\theta - z sin\theta, r sin\theta + z cos \theta) dz} $


$ = \delta(r) $


$ {\color{green} \text{Here, the student uses the intuitive solution: in this case the answer does not depend on } \theta \text{, since the image just contains a peak at origin. } } $


$ \color{blue}\text{b) Calculate the forward projection }p_{\theta}(r) \text{, for } f(x,y) = \delta(x-1,y-1). $

$ \color{blue}\text{Solution 1:} $

$ p_{\theta}(r) = \int_{-\infty}^{+\infty}{\delta(r cos\theta - z sin\theta - 1, r sin\theta + z cos \theta - 1) dz} $

$ = z \text{ when } \left\{\begin{matrix} r cos \theta - z sin \theta = 1 \\ r sin \theta + z cos \theta = 1 \end{matrix}\right. $


$ = \frac{r cos\theta - 1}{sin \theta}, \theta > 0 $

$ {\color{red} \text{This answer is incorrect. The correct answer is as following:}} $

$ \color{green} \text{ Similar to the solution 1 to part a) we define u: } u = r cos\theta - z sin\theta - 1 $


$ \color{green} \text{ Following the same logic as in part a) we obtain the final answer:} $


$ \color{green} p_{\theta}(r) = \delta(r - (cos\theta + sin \theta)) = \delta(r - \sqrt{2} cos (\theta - \frac{\pi}{4})) $


$ \color{blue}\text{Solution 2:} $

QE 11 CS5 2 b sol2.PNG

$ \tilde{p}_\theta(r) = p_{\theta}(r - \sqrt{1+1} cos(\theta - tan^{-1}(\frac{1}{1}))) $


$ = p_\theta(r - \sqrt{2} cos(\theta - \frac{\pi}{4})) $


$ = \delta(r - \sqrt{2} cos(\theta - \frac{\pi}{4})) $


$ {\color{green} \text{Again, the student uses the intuitive solution: in this case the answer does depend on } \theta \text{, since the peak is shifted from the origin to the point } (1,1). } $



$ \color{blue}\text{c) Calculate the forward projection }p_{\theta}(r) \text{, for } f(x,y) = rect \left(\sqrt[]{x^2+y^2} \right). $


$ \color{blue}\text{Solution 1:} $


$ p_{\theta}(r) = \int_{-\infty}^{+\infty}{rect(\sqrt{(r cos\theta - z sin\theta)^2 + (r sin\theta + z cos \theta)^2)} dz} $


$ \color{green} \text{Recall should be added:} $

$ \color{green} rect(t) = \left\{\begin{matrix} 1, for |t|\leq \frac{1}{2} \\ 0, otherwise \end{matrix}\right. $


$ \color{green} \text{therefore: } $


$ p_{\theta}(r) = \int_{-\sqrt{\frac{1}{4} - r^2}}^{\sqrt{\frac{1}{4} - r^2}}{1 dz} $


$ = \left\{\begin{matrix} \sqrt{1 - 4r^2}, &\text{ if }|r| \leq \frac{1}{2} \\ 0, &\text{ otherwise} \end{matrix}\right. $


$ \color{blue}\text{Solution 2:} $

QE 11 CS5 2 c sol2.PNG

$ p_{\theta}(r) = \int_{-\infty}^{+\infty}{f(r cos\theta - z sin\theta, r sin\theta + z cos \theta) dz} $


$ = \int_{-\sqrt{\frac{1}{4} - r^2}}^{\sqrt{\frac{1}{4} - r^2}}{1 dz} = \sqrt{1 - 4r^2}, \text{ if }|r| \leq \frac{1}{2} $


$ \text{ else } p_{\theta}(r) = 0 $


$ \color{blue}\text{d) Calculate the forward projection }p_{\theta}(r) \text{, for } f(x,y) = rect \left(\sqrt[]{(x-1)^2+(y-1)^2} \right). $

$ \color{blue}\text{Solution 1:} $


$ p_{\theta}(r) = \int_{-\infty}^{+\infty}{rect \left( \sqrt{(r cos\theta - z sin\theta - 1)^2 + (r sin\theta + z cos \theta - 1)^2} \right) dz} $


$ = \int_{-\sqrt{\frac{1}{4} - (r - (cos\theta + sin\theta))^2}}^{\sqrt{\frac{1}{4} - (r - (cos\theta + sin\theta))^2}}{1 dz} $


$ = \left\{\begin{matrix} \sqrt{1 - 4(r - (cos\theta + sin\theta))^2}, &\text{ if }|r| \leq \frac{1}{2} \\ 0, &\text{ otherwise} \end{matrix}\right. $


$ \color{green} \text{ To make it more clear, the following form could be obtained:} $

$ \color{green} = \left\{\begin{matrix} \sqrt{1 - 4(r - \sqrt{2} cos (\theta - \frac{\pi}{4}))^2}, &\text{ if }|r| \leq \frac{1}{2} \\ 0, &\text{ otherwise} \end{matrix}\right. $


$ \color{blue}\text{Solution 2:} $

$ \tilde{p}_\theta(r) = p_{\theta}(r - \sqrt{1+1} cos(\theta - tan^{-1}(\frac{1}{1}))) $


$ = p_\theta(r - \sqrt{2} cos(\theta - \frac{\pi}{4})) $


$ \text{ where } p_\theta(r) = \left\{\begin{matrix} \sqrt{1 - 4r^2}, &\text{ if }|r| \leq \frac{1}{2} \\ 0, &\text{ else} \end{matrix}\right. $


$ {\color{green} \text{Here, the student uses the results from solutions to part b and c.} } $


$ \color{blue}\text{e) Describe in precise detail, the steps required to perform filtered back projection (FBP) reconstruction of } f(x,y). $


$ \color{blue}\text{Solution 1:} $

$ 1. \text{ Compute } \rho_{\theta}(r) $


$ 2. \text{ Compute FT of step 1.} $

$ 3. \text{ Multiply step 2 by the filter } H(\rho) = |\rho| = f_c \left [ rect(\frac{f}{2f_c}) - \Lambda(\frac{f}{f_c}) \right ] $

$ 4. \text{ Compute inverseFT of step 3.} $

$ \color{red} \text{ This answer is not complete. We need the final step:} $


$ \color{red} 5. \text{ Back project } g_{\theta}(r) \text{ (obtained from step 4) and get: } $

$ \color{red} f(x,y) = \int_{0}^{\pi}{g_\theta(xcos\theta + ysin\theta)d\theta} $


$ \color{blue}\text{Solution 2:} $

$ 1. \text{ Measure the projections } \rho_{\theta}(r) \text{ at various angles} $

$ 2. \text{ Filter the projections } \rho_{\theta}(r) \text{ with } h(r) \text{, where } H(\rho) = |\rho| \text{ and get } g_{\theta}(r) $

$ 3. \text{ Back project } g_{\theta}(r) \text{ along } r = xcos\theta + ysin\theta \text{ and get } $

$ f(x,y) = \int_{0}^{\pi}{g_\theta(xcos\theta + ysin\theta)d\theta} $


Solution 3:

a)

$ p_\theta(r)=\begin{cases}1, & \text{if } r=0\\0, & \text{otherwisr}\end{cases}=\delta(r) $.

CW QE 2011 CS5 2 a sol3.png


b)

  • If $ 0\leq\theta<\frac{\pi}{4} $, $ d=\sqrt{2}\text{cos}(\frac{\pi}{4}-\theta)=\sqrt{2}[\text{cos}\frac{\pi}{4}\text{cos}\theta+\text{sin}\frac{\pi}{4}\text{sin}\theta]=\text{cos}\theta+\text{sin}\theta $.

        CW QE 2011 CS5 2 b 1 sol3.png

  • If $ \frac{\pi}{4}\leq \theta < \frac{\pi}{2} $, $ d=\sqrt{2}\text{cos}(\theta-\frac{\pi}{4})=\sqrt{2}\text{cos}(\frac{\pi}{4}-\theta)=\text{cos}\theta+\text{sin}\theta $.

        CW QE 2011 CS5 2 b 2 sol3.png

  • If $ \frac{\pi}{2}\leq \theta < \frac{3\pi}{4} $, $ d=\sqrt{2}\text{sin}(\frac{\pi}{4}-(\theta-\frac{\pi}{2}))=\sqrt{2}\text{sin}(\frac{3\pi}{4}-\theta)=\sqrt{2}[\text{sin}\frac{3\pi}{4}\text{cos}\theta-\text{cos}\frac{3\pi}{4}\text{sin}\theta]=\text{cos}\theta+\text{sin}\theta $.

        CW QE 2011 CS5 2 b 3 sol3.png

  • If $ \frac{3\pi}{4}\leq \theta < 2\pi $, $ d=-\sqrt{2}\text{sin}(\theta-\frac{\pi}{2}-\frac{\pi}{4})=-\sqrt{2}\text{sin}(\theta-\frac{3\pi}{4})=-\sqrt{2}[\text{sin}\theta\text{cos}\frac{3\pi}{4}-\text{cos}\theta\text{sin}\frac{3\pi}{4}]=\text{cos}\theta+\text{sin}\theta $.

        CW QE 2011 CS5 2 b 4 sol3.png

$ \Rightarrow p_\theta(r)=\delta(r-(\text{cos}\theta+\text{sin}\theta)) $ for all $ \theta $.


c)

Since $ f(x,y) $ is symmetric to $ (0,0) $

$ \Rightarrow p_\theta(r)=p_0(r)=\int_{-\sqrt{\frac{1}{4}-r^2}}^{\sqrt{\frac{1}{4}-r^2}}1\cdot dz=2\int_0^{\sqrt{\frac{1}{4}-r^2}}1\cdot dz\text{, for }\vert r\vert<\frac{1}{2} $

CW QE 2011 CS5 2 c sol3.png

$ \Rightarrow p_\theta(r)=2\sqrt{\frac{1}{4}-r^2}\text{rect}(r) $


d)

From part (b), we know that, if $ f(x,y) $ has $ (1,1) $ shift in $ x-y $ plane, the $ p_\theta(r) $ would have $ (\text{cos}\theta+\text{sin}\theta) $ shift in $ r $-direction.

$ \Rightarrow p_\theta(r)=2\sqrt{\frac{1}{4}-(r-(\text{cos}\theta+\text{sin}\theta))^2}\text{rect}(r-(\text{cos}\theta+\text{sin}\theta)) $


e)

  • First, measure the forward projection $ p_\theta(r) $ for sufficient angles $ \theta s $.
  • Second, filter the forward projection with $ h(r): g_\theta(r)=h(r)\ast p_\theta(r) $, where $ h(r)=\text{CTFT}^{-1}\left\{\vert\rho\vert\right\} $.
  • Finally, back project filtered projection to reconstruct $ f(x,y) $. $ f(x,y)=\int_0^\pi g_\theta(x\text{cos}\theta+y\text{sin}\theta)d\theta $.


Solution 4:

a)

$ P_\theta(\rho)=F(\rho\text{cos}\theta, \rho\text{sin}\theta) $

When $ f(x,y)=\delta(x,y) $, $ F(\mu,\nu)=1 $.

So $ p_\theta(r)=\delta_\theta(r) $.

Comments:

  • The answer would be easier to understand if the last statement was expressed in this way: $ p_{\theta}(r)=\mathcal{F}^{-1}\{ P_{\theta}(\rho)\}=\mathcal{F}^{-1}\{1\}=\delta_{\theta}(r) $, where $ \mathcal{F}^{-1}\{\cdot\} $ is the Fourier inverse transform.


b)

When $ f(x,y)=\delta(x-1,y-1) $, $ F(\mu,\nu)=e^{-j\mu}\cdot e^{-j\nu} $.

So $ P_\theta(\rho)=e^{-j\rho\text{cos}\theta}\cdot e^{-j\rho\text{sin}\theta} $, $ p_\theta(r)=\delta(r-\text{cos}\theta-\text{sin}\theta) $.


c)

CW QE 2011 CS5 2 c sol4.png

$ p_\theta(r)=\int_{-\sqrt{\frac{1}{4}-r^2}}^{\sqrt{\frac{1}{4}-r^2}}1dz=\sqrt{1-4r^2} $

So $ p_\theta(r)=\begin{cases} \sqrt{1-4r^2}, & |r|<\frac{1}{2}\\0, & |r|>\frac{1}{2}\end{cases} $.


d)

CW QE 2011 CS5 2 d sol4.png

$ r_0=\sqrt{2}\cdot\left(\frac{\pi}{4}-\theta\right) $

$ p_\theta(r)=\int_{-\infty}^{\infty}f(r\text{cos}\theta-z\text{sin}\theta, r\text{sin}\theta+z\text{cos}\theta)dz=\int_{-\sqrt{\frac{1}{4}-(r-r_0)^2}}^{\sqrt{\frac{1}{4}-(r-r_0)^2}}adz $

So $ p_\theta(r)=\begin{cases}\sqrt{1-4\left(r-\sqrt{2}\left(\frac{\pi}{4}-\theta\right)\right)}, & |r-r_0|\leq\frac{1}{2}\\0, & \text{otherwise}\end{cases} $.

Comments:

  • The computation for $ r_0 $ is wrong. The correct expression of $ r_0 $ should be: $ r_0 = \sqrt{2}\sin(\frac{\pi}{4}-\theta) $. It can also be expressed as: $ r_0=\cos\theta+\sin\theta $.
  • The rest statement is correct, and then the final result becomes: $ p_{\theta}(r)=\begin{cases}\sqrt{1-4(r-r_0)}, & |r-r_0|\leq\frac{1}{2}\\0, & \text{otherwise} \end{cases} $.


e)

$ \begin{align} f(x,y) & =\int_{-\infty}^{\infty}\int_{-\infty}^{\infty}F(\mu,\nu)e^{2\pi j(x\mu+y\nu)}d\mu d\nu\\ & = \int_{-\infty}^{\infty}\int_{0}^{\pi}p_\theta(\rho)e^{2\pi j(x\rho\text{cos}\theta+y\rho\text{sin}\theta)}|\rho|d\theta d\rho\\ & = \int_{0}^{\pi}\underbrace{\int_{-\infty}^{\infty}|\rho|p_\theta(\rho)e^{2\pi j(x\rho\text{cos}\theta+y\rho\text{sin}\theta)}d\rho}_{g_\theta(x\text{cos}\theta+y\text{sin}\theta)}d\theta \end{align} $

So $ g_\theta(t) $ is given by:

$ g_\theta(t)=\int_{-\infty}^{\infty}|\rho|p_\theta(\rho)e^{2\pi j\rho t}d\rho=\text{CTFT}^{-1}\{|\rho|p_\theta(\rho)\}=h(t)\ast p_\theta(r) $

So what we do is:

  1. measure projection $ p_\theta(r) $;
  2. filter the projection $ g_\theta(r)=h(r)\ast p_\theta(r) $;
  3. back project filtered projection: $ f(x,y)=\int_0^{\pi}g_\theta(x\text{cos}\theta)+y\text{sin}\theta)d\theta $.



Related Problems

a) Calculate the forward projection $ p_\theta(r) $, for $ f(x,y)=\text{rect}(x,y) $, as $ \theta=0 $ and $ \theta=\pi/4 $.

b) Calculate the forward projection $ p_\theta(r) $, for $ f(x,y)=\delta(x-a,y-b) $.


"Communication, Networks, Signal, and Image Processing" (CS)- Question 5, August 2011

Go to


Back to ECE Qualifying Exams (QE) page

Alumni Liaison

To all math majors: "Mathematics is a wonderfully rich subject."

Dr. Paul Garrett